LSAT and Law School Admissions Forum

Get expert LSAT preparation and law school admissions advice from PowerScore Test Preparation.

 Administrator
PowerScore Staff
  • PowerScore Staff
  • Posts: 8919
  • Joined: Feb 02, 2011
|
#27394
Complete Question Explanation

Resolve the Paradox—#%. The correct answer choice is (C)

The paradox in the argument is that the provinces and states that have more stringent safety requirements also have higher average rates of accidents. Even so, experts agree that the more stringent requirements actually are effective. This type of “surprisingly low/high rate of success” scenario has appeared in a number of Resolve the Paradox questions, including the following:
  • An anti-theft device is known to reduce theft, but cars using the anti-theft device are stolen at a higher rate than cars without the device.

    Explanation: The device is placed on highly desirable cars that are prone to being stolen, and the device actually lessens the rate at which they are stolen.

    A surgeon has a low success rate while operating, but the director of the hospital claims the surgeon is the best on the staff.

    Explanation: The surgeon operates on the most complex and challenging cases.

    A bill collector has the lowest rate of success in collecting bills, but his manager claims he is the best in the field.

    Explanation: The bill collector is assigned the toughest cases to handle.
These scenarios underscore the issue present in the question: other factors in the situation make it more difficult to be successful. With the car safety requirements, you should look for an answer that shows that there is a situation with the roads that affects the accident rates. A second possible explanation is that the seat belts are not actually used by a majority of drivers and the safety inspections are not made or are rubber-stamp certifications. This answer is less likely to appear because it is fairly obvious.

Answer choice (A): The stimulus specifies that annual safety inspections—regardless of what is examined—are already in place. Therefore, this answer does not explain why the average rate of accidents is higher in those states.

Answer choice (B): Assuming that overconfidence leads to accidents, the answer could support the assertion that states with more stringent requirements have higher accident rates. But, this answer would also suggest that the experts are wrong in saying that more stringent standards reduce accident rates, so this answer cannot be correct.

Answer choice (C): This is the correct answer choice, and the answer conforms to the discussion above. If the roads are generally more dangerous, then the stringent requirements could reduce the accident rate while at the same time the accident rate could remain relatively high. Since this scenario allows all sides of the situation to be correct and it explains how the situation could occur, this is the correct answer.

Answer choice (D): This answer supports only one side of the paradox. The answer confirms that the experts are correct, but it does not explain why these provinces have higher accident rates. Thus, it does not resolve the paradox.

Answer choice (E): This answer appears attractive at first, but the number of miles of roadway in the provinces is irrelevant because the stimulus specifically references “accidents per kilometer driven.” Since the accident rate is calculated as per-miles-driven, the actual number of miles of roadway is irrelevant.
User avatar
 lemonade42
  • Posts: 11
  • Joined: Feb 23, 2024
|
#105481
Hello,

I understand why A,B,D, and E are all incorrect and why C is correct. But at first, I didn't believe that C was correct as well because it said "stringent car safety programs" instead of what the stimulus used which was "stringent car safety requirements". So I thought it doing some sort of shell game thing. And I thought programs and requirements wouldn't match each other because programs seem like classes or lessons and requirements are more like rules of required seat belt use and inspections. How can we connect the two?
User avatar
 Jeff Wren
PowerScore Staff
  • PowerScore Staff
  • Posts: 385
  • Joined: Oct 19, 2022
|
#105498
Hi lemonade,

I'm not sure if the copy of the question that you are reading has an error, but I'm looking at two different copies of the question (including the copy published by LSAC), and Answer C states "stringent car safety requirements" just like the stimulus. Programs are not mentioned anywhere in this problem.

If you did misread the answer, use this as a learning opportunity. Better that it happens while studying, then on the real test!

Get the most out of your LSAT Prep Plus subscription.

Analyze and track your performance with our Testing and Analytics Package.